Ableitung des Photonenpropagators

Dies wurde schon früher gefragt (siehe Ableitung des Photonenpropagators ), aber beim Ableiten des Photonenpropagators kommen wir zu:

[ G μ v k 2 + a 1 a k μ k v ] D ~ v λ ( k ) = δ λ μ

Wir sollen den Operator auf der linken Seite umkehren, um den Propagator zu erhalten. Ich weiß, dass wir den Ansatz verwenden sollen:

D ~ v λ ( k ) = A G v λ + B k v k λ

um die Koeffizienten A und B zu bestimmen. Aber brauchen wir nicht zwei Gleichungen, um zwei Variablen zu bestimmen? Wenn ja, wie lautet die zweite Gleichung?

Auch ich konnte mit dieser Bedingung oben nicht sehr weit kommen, alles was ich bekam war:

3 A A + B a = 4 k 2

Ist das überhaupt richtig? Wenn ja, wie weiter vorgehen? Eine detaillierte Lösung wäre sehr dankbar.

Wenn Sie sagen, dass es schon einmal gefragt wurde, meinen Sie das: physical.stackexchange.com/questions/137577/… es ist eine gute Idee, den Link, auf den Sie sich beziehen, in Ihren Beitrag einzufügen, danke
Keine Notwendigkeit für eine Entschuldigung mit meiner Art von Bemerkungen, Sie müssen sich nur entschuldigen, wenn/wenn Sie auf persönlicher Ebene wirklich etwas falsch machen :), viel Glück mit Ihrer Frage.

Antworten (2)

Als Originalposter dieser Frage denke ich, dass ich vielleicht helfen kann. Ich werde die gleiche Notation wie in der ursprünglichen Frage verwenden.

Ausgehend von der Gleichung

(1) ( k 2 G μ v + ( 1 1 ξ ) k μ k v ) D v ρ ( k ) = ich δ μ ρ

Wir machen den Ansatz

(2) D μ ρ ( k ) = A G μ ρ + B k μ k ρ .

Einsetzen dieses Ansatzes in die Gl. ( 1 ) wir bekommen

(3) [ k 2 G μ v + ( 1 1 ξ ) k μ k v ] [ A G v ρ + B k v k ρ ] = ich δ μ ρ .

(Dir fehlt der Faktor von ich von der rechten Seite ( 3 ) in Ihrer Frage, aber ich werde es hier setzen.)

Was Sie tun müssen, ist nicht zu lösen A Und B durch Isolation, sondern vergleiche tatsächlich die Koeffizienten auf beiden Seiten von Gl. ( 3 ) . Erweitern Sie das Produkt und nach ein wenig Algebra sollten Sie ankommen

A = ich k 2 , B = ich k 4 ( 1 ξ ) ,

was Ihnen die Koeffizienten für die Umkehrung gibt ( 2 ) .

Ich sehe jetzt, dass ich es völlig falsch verstanden habe. Vielen Dank für die ausführliche Antwort.

Da Sie Tensoren mit griechischen Indizes verwenden, möchte ich darauf hinweisen, dass die Konvention dies impliziert D ¯ v λ ( k ) = A G v λ + B k v k λ sind technisch gesehen 16 Gleichungen, was mehr als genug ist, um sie zu isolieren A Und B . Wenn G v λ ist die Metrik und k v ein Impulstensor ohne Operatorwert ist, sollten beide Terme in der rechten Seite symmetrisch sein, was bedeutet, dass Sie höchstens 10 Gleichungen haben (weniger überbestimmt ist besser).

Theoretisch sollten Sie in der Lage sein, sich zu isolieren A Und B durch Eingabe der verschiedenen Komponenten von G Und k . Da ich nicht weiß, woraus diese Tensoren bestehen, kann ich da nicht helfen. Aber ich kann Ihnen sagen, dass es nicht darum geht, zu wenige Gleichungen zu haben (tatsächlich wäre ich verärgert, wenn ich zu viele Gleichungen hätte).